MGMAT 700-800

This topic has expert replies
Master | Next Rank: 500 Posts
Posts: 371
Joined: Fri Mar 06, 2009 2:48 am
Thanked: 27 times
GMAT Score:740

MGMAT 700-800

by 2010gmat » Sun Nov 15, 2009 7:32 am
Recently in City X, developers have stopped buying land, contractors have found themselves going without work for longer periods, and banks have issued fewer mortgages. There must be fewer new residents moving to City X than there were previously.

Which of the following indicates a flaw in the reasoning above?

A This year several housing blocks have gone on the market after being held up for months by legal red tape.
B The average size of a new home has increased significantly over the past several years.
C Re-sales of condominiums have increased over the past six months.
D The cost of materials such as lumber and cement has decreased over the past year.
E Sales of other big-ticket items, such as automobiles and boats, has remained steady over the past year.

OA to follow....i somehow found the OE weird....

Legendary Member
Posts: 2326
Joined: Mon Jul 28, 2008 3:54 am
Thanked: 173 times
Followed by:2 members
GMAT Score:710

by gmatmachoman » Sun Nov 15, 2009 10:07 am
IMO A

I shall post my explanation if I am right.

Master | Next Rank: 500 Posts
Posts: 189
Joined: Tue Sep 16, 2008 10:42 am
Thanked: 11 times
Followed by:1 members

by hitmewithgmat » Sun Nov 15, 2009 12:23 pm
The conclusion of the argument is that fewer new residents move to city X based on 3 premises.
1. contractors stop buying land
2. no work to do
3. fewer mortage

The conclusion is based on these 3 premises.

In other words, 3 premises causes the conclusion and the author assumes that there are no other possibilities to explain this situation. But, we need to weaken the argument so there must be other alternate possibility that we can find. Namely, while these(3 premises) may be true, it is also possible that there are other causes. (Remember, weaken-the-argument question is really asking you to find a hole(missing link) in the argument.)

Let's find an alternative. That's what (A) says. Not because of 3 premises, but because of "unsold houses" or "avaiable houses" on the market.

(B)"size" of the house? it's out of the scope.
(C)(D) and (E) are irrelevant.

Therefore, the answer is (A) for sure.

Hope this helps.
Disclaimer-I am not a GMAT savvy yet, but I am learning everyday with my fellow beatthegmat citizens.

I AM DETERMINED TO CRASH/NIX OUT/ATTACK BRUTALLY/CRACK VERBAL PART OF GMAT. ROAR!

Master | Next Rank: 500 Posts
Posts: 371
Joined: Fri Mar 06, 2009 2:48 am
Thanked: 27 times
GMAT Score:740

by 2010gmat » Sun Nov 15, 2009 11:51 pm
thnxx oa is a....i think i got too critical...the key to successfully beat the CR ques is not to get too critical...;)...was stuck between a and c

Legendary Member
Posts: 2326
Joined: Mon Jul 28, 2008 3:54 am
Thanked: 173 times
Followed by:2 members
GMAT Score:710

by gmatmachoman » Mon Nov 16, 2009 9:22 am
well said HitmewithGMAT...


Yeah one way of weakening the conclusion is to find a alternate cause for which the event has occured.

A states that alternate cause and does weakens the assumption and thereby the conclusion!

Master | Next Rank: 500 Posts
Posts: 200
Joined: Sat Aug 22, 2015 10:27 am

by james33 » Sun May 15, 2016 10:05 pm
I like the explanation on A